Diễn Đàn MathScopeDiễn Đàn MathScope
  Diễn Đàn MathScope
Ghi Danh Hỏi/Ðáp Thành Viên Social Groups Lịch Ðánh Dấu Ðã Ðọc

Go Back   Diễn Đàn MathScope > Sơ Cấp > Việt Nam và IMO > 2014

News & Announcements

Ngoài một số quy định đã được nêu trong phần Quy định của Ghi Danh , mọi người tranh thủ bỏ ra 5 phút để đọc thêm một số Quy định sau để khỏi bị treo nick ở MathScope nhé !

* Nội quy MathScope.Org

* Một số quy định chung !

* Quy định về việc viết bài trong diễn đàn MathScope

* Nếu bạn muốn gia nhập đội ngũ BQT thì vui lòng tham gia tại đây

* Những câu hỏi thường gặp

* Về việc viết bài trong Box Đại học và Sau đại học


Trả lời Gởi Ðề Tài Mới
 
Ðiều Chỉnh Xếp Bài
Old 03-01-2014, 11:27 AM   #1
huynhcongbang
Administrator

 
huynhcongbang's Avatar
 
Tham gia ngày: Feb 2009
Đến từ: Ho Chi Minh City
Bài gởi: 2,413
Thanks: 2,165
Thanked 4,188 Times in 1,381 Posts
Gửi tin nhắn qua Yahoo chát tới huynhcongbang
[VMO 2014] Bài 2 - Đại số

Bài 2.
Cho đa thức $P(x)={{({{x}^{2}}-7x+6)}^{2n}}+13$ với $n$ là số nguyên dương. Chứng minh rằng đa thức $P(x)$ không thể biểu diễn được dưới dạng tích của $n+1$ đa thức khác hằng số với hệ số nguyên.
[RIGHT][I][B]Nguồn: MathScope.ORG[/B][/I][/RIGHT]
 
__________________
Sự im lặng của bầy mèo
huynhcongbang is offline   Trả Lời Với Trích Dẫn
Old 03-01-2014, 11:51 AM   #2
hakudoshi
+Thành Viên+
 
hakudoshi's Avatar
 
Tham gia ngày: Feb 2012
Đến từ: vật chất->sự sống->tư duy->cảm xúc->???
Bài gởi: 210
Thanks: 102
Thanked 179 Times in 90 Posts
$P(x)>0$ nên $P(x)$ vô nghiệm. Do đó $n+1$ đa thức kia có bậc không bé hơn 2 và đều vô nghiệm.
Mặt khác $P(1)=P(6)=13$. ... Bí
[RIGHT][I][B]Nguồn: MathScope.ORG[/B][/I][/RIGHT]
 
__________________
Touch me touch me, don't be shy
I'm in charge like a G.U.Y.
I'll lay down face up this time
Under you like a G.U.Y.
hakudoshi is offline   Trả Lời Với Trích Dẫn
Old 03-01-2014, 12:08 PM   #3
DogLover
+Thành Viên+
 
Tham gia ngày: Jan 2014
Bài gởi: 13
Thanks: 9
Thanked 12 Times in 7 Posts
Lâu lắm mới thấy một bài đa thức khả quy.
Giả sử có thể phân tích được thành tích của $n+1$ đa thức $P_{i}(x)$ khác hằng hệ số nguyên.
Nhận xét 1: P(x) vô nghiệm do đó các đa thức $P_{i}(x)$ đều có bậc chẵn. Và vì P(x) có bậc 4n nên suy ra có ít nhất 2 chỉ số $i$ sao cho $P_{i}(x)$ có bậc 2, giả sử là $k$ và $l$.
Nhận xét 2: Với mọi $i$ thì $P_{i}(6)-P_{i}(1)$ chia hết cho 5

Cho $x=1$ và $x=6$ thì ta thu được $$P_{1}(1).P_{2}(1)...P_{n+1}(1) = P_{1}(6).P_{2}(6)...P_{n+1}(6) = 13$$
Suy ra $P_{i}(1)$ và $P_{i}(6)$ chỉ nhận giá trị trong ${-1,1,13,-13}$.
Mà $P_{i}(6)-P_{i}(1)$ chia hết cho 5 nên với mọi i thì $P_{i}(1) = P_{i}(6)$

Xét $i = k$ và $i = l$, rõ ràng không thể có trường hợp $|P_{k}(1)| = |P_{k}(6)|=|P_{l}(1)| = |P_{l}(6)| = 13$ nên hoặc $|P_{k}(1)| = |P_{k}(6)| =1$ hoặc $|P_{l}(1)| = |P_{l}(6)| =1$. Giả sử là $P_{k}$.
Khi đó có hai trường hợp xảy ra là $P_{k}(x) = (x-1)(x-6)+1$ hoặc $P_{k}(x) = (x-1)(x-6)-1$. Cả hai trường hợp này $P_{k}(x)$ đều có nghiệm, suy ra P(x) có nghiệm ==> vô lý.
Như vậy điều giả sử là sai. Và bài toán đã được chứng minh
[RIGHT][I][B]Nguồn: MathScope.ORG[/B][/I][/RIGHT]
 

thay đổi nội dung bởi: DogLover, 03-01-2014 lúc 12:11 PM
DogLover is offline   Trả Lời Với Trích Dẫn
The Following User Says Thank You to DogLover For This Useful Post:
huynhcongbang (03-01-2014)
Old 03-01-2014, 12:10 PM   #4
hien123
+Thành Viên+
 
Tham gia ngày: Sep 2010
Đến từ: THPT chuyên Phan Bội Châu, Nghệ An
Bài gởi: 353
Thanks: 19
Thanked 261 Times in 165 Posts
Xét P(x+1) rồi dùng tiêu chuẩn eisteiner mở rộng ta suy ra có đúng n đa thức bậc 2 va 1 đa thức bậc 2n. Nhận xét mỗi đa thức bậc 2 phải có hệ số tự do chia hết cho 11. Suy ra $36^n+13$ chia hết cho $11^n$, mâu thuẫn.
[RIGHT][I][B]Nguồn: MathScope.ORG[/B][/I][/RIGHT]
 
__________________
$z=\left | z \right |e^{i\varphi } $
hien123 is offline   Trả Lời Với Trích Dẫn
The Following 3 Users Say Thank You to hien123 For This Useful Post:
batigoal (03-01-2014), huynhcongbang (03-01-2014), thaygiaocht (03-01-2014)
Old 03-01-2014, 12:20 PM   #5
DaiToan
+Thành Viên+
 
Tham gia ngày: Oct 2010
Đến từ: THPT Chuyên Vĩnh Phúc
Bài gởi: 280
Thanks: 29
Thanked 361 Times in 123 Posts
Giả sử P(x) phân tích được thành tích của (n+1) đa thức khác hằng hệ số nguyên.
Vì P(x) vô nghiệm thực nên các nhân tử đó phải bậc chẵn. Nhưng bậc của P(x) bằng 4n nên phải tồn tại một nhân tử f(x) bậc 2 (có lẽ đây là mấu chốt!)
Có thể giả sử f(x) có hệ số của x2 là 1: $$f(x) = {x^2} + bx + c;{\rm{ }}b,c \in $ $. Vì f(x) vô nghiệm thực nên c>0.
Do $$\mathop {\lim }\limits_{x \to + \infty } f(x) = + \infty $ $ và f(x) vô nghiệm nên f(1)>0; f(6)>0.
Mà P(1)=P(6)=13 nên f(1), f(6) là ước dương của 13;
Nhưng f(6)-f(1) chia hết cho 5 nên f(1)=f(6)=1 hoặc 13
Từ f(1)=f(6) suy ra b=-7, suy ra $$f(x) = {x^2} - 7x + c$ $.
* Nếu f(1)=1 thì c=7, suy ra $$f(x) = {x^2} - 7x + 7$ $ có nghiệm thực (vô lí).
* Nếu f(1)=13 thì c=19, suy ra $$f(x) = {x^2} - 7x + 19$ $.
Ta có f(2)=9 và $$P(2) = {4^{2n}} + 13$ $.
Rõ ràng P(2) không chia hết cho 3 trong khi f(2) chia hết cho 3 (vô lí).
Vậy bài toán được chứng minh hoàn toàn.
[RIGHT][I][B]Nguồn: MathScope.ORG[/B][/I][/RIGHT]
 

thay đổi nội dung bởi: DaiToan, 03-01-2014 lúc 12:22 PM Lý do: Tự động gộp bài
DaiToan is offline   Trả Lời Với Trích Dẫn
The Following 4 Users Say Thank You to DaiToan For This Useful Post:
phamhuudanh (03-01-2014), ruud (03-01-2014), thaygiaocht (03-01-2014), Trànvănđức (04-01-2014)
Old 03-01-2014, 02:03 PM   #6
quangvinht2
+Thành Viên+
 
Tham gia ngày: Feb 2009
Bài gởi: 18
Thanks: 9
Thanked 25 Times in 8 Posts
Vì P(x) luôn dương nên không có ước bậc lẻ. Giả sử P(x) phân tích thành tích n+1 đa thức monic bậc chẵn có tổng bậc là 4n nên tồn tại hai ước P1(x) và P2(x) bậc hai bất khả quy trên R. Vì cả hai monic nên luôn dương và P1(1).P2(1)|13, P1(6)P2(6)|13 nên chỉ có mấy khả năng sau
1/ P1(1)=P1(6)=1=> P1(x) có nghiệm thực (mâu thuẫn)
2/{P1(1);P1(6)}={1;13} => mâu thuẫn vì P1(6)-P1(1) là bội 5.
3/ P1(1)=P1(6)=13 thì P2(1)=P2(6)=1 mâu thuẫn giống phần 1.
[RIGHT][I][B]Nguồn: MathScope.ORG[/B][/I][/RIGHT]
 
quangvinht2 is offline   Trả Lời Với Trích Dẫn
Old 03-01-2014, 02:53 PM   #7
huynhcongbang
Administrator

 
huynhcongbang's Avatar
 
Tham gia ngày: Feb 2009
Đến từ: Ho Chi Minh City
Bài gởi: 2,413
Thanks: 2,165
Thanked 4,188 Times in 1,381 Posts
Gửi tin nhắn qua Yahoo chát tới huynhcongbang
Xin giới thiệu với mọi người lời giải của anh Cẩn, ý tưởng biến đổi đại số kết hợp với tính chất số học khá thuần túy.




[RIGHT][I][B]Nguồn: MathScope.ORG[/B][/I][/RIGHT]
 
Hình Kèm Theo
Kiểu File : jpg 1514987_10202518010368424_1997443922_n.jpg (55.7 KB, 437 lần tải)
Kiểu File : jpg 1533766_10202518010048416_1392480634_n.jpg (29.1 KB, 411 lần tải)
__________________
Sự im lặng của bầy mèo
huynhcongbang is offline   Trả Lời Với Trích Dẫn
The Following 3 Users Say Thank You to huynhcongbang For This Useful Post:
Kém Toán (03-01-2014), luugiangnam (03-01-2014), phamhuudanh (03-01-2014)
Old 03-01-2014, 02:58 PM   #8
thaygiaocht
+Thành Viên+
 
thaygiaocht's Avatar
 
Tham gia ngày: Aug 2012
Đến từ: Chuyên Hà Tĩnh
Bài gởi: 165
Thanks: 793
Thanked 216 Times in 93 Posts
Đến đoạn $P(1)=P(6)$ rồi như các lời giải ở trên rồi có thể viết $P(x)=(x-1)(x-6)+k$. Do $P(x)$ vô nghiệm, giải $\Delta >0$ được $k>\dfrac{25}{4}$, lại có $k|13$ nên $k=13$.
Thay vào $P(x)=x^2-7x+19$.
Do đó $6^{2n}=-6 (\mod 19)$, có thể chỉ ra điều này vô lý bằng quy nạp.
[RIGHT][I][B]Nguồn: MathScope.ORG[/B][/I][/RIGHT]
 
__________________
https://www.facebook.com/thaygiaocht

thay đổi nội dung bởi: thaygiaocht, 03-01-2014 lúc 05:41 PM
thaygiaocht is offline   Trả Lời Với Trích Dẫn
The Following 2 Users Say Thank You to thaygiaocht For This Useful Post:
hoangqnvip (03-01-2014), huynhcongbang (03-01-2014)
Old 03-01-2014, 04:22 PM   #9
huynhcongbang
Administrator

 
huynhcongbang's Avatar
 
Tham gia ngày: Feb 2009
Đến từ: Ho Chi Minh City
Bài gởi: 2,413
Thanks: 2,165
Thanked 4,188 Times in 1,381 Posts
Gửi tin nhắn qua Yahoo chát tới huynhcongbang
Bài 2 này có nét gì đó giống một bài cũ trong IMO Shortlist 1991.

http://www.artofproblemsolving.com/F...6d360#p1225026

Cũng kiểu đa thức nguyên và yêu cầu chứng minh chặn trên cho số thừa số mà nó phân tích ra được.
[RIGHT][I][B]Nguồn: MathScope.ORG[/B][/I][/RIGHT]
 
__________________
Sự im lặng của bầy mèo
huynhcongbang is offline   Trả Lời Với Trích Dẫn
The Following 2 Users Say Thank You to huynhcongbang For This Useful Post:
hoangqnvip (03-01-2014), thaygiaocht (03-01-2014)
Old 04-01-2014, 08:33 AM   #10
namdung
Administrator

 
Tham gia ngày: Feb 2009
Đến từ: Tp Hồ Chí Minh
Bài gởi: 1,343
Thanks: 209
Thanked 4,066 Times in 778 Posts
Gửi tin nhắn qua Yahoo chát tới namdung
Thực ra ta có thể chứng minh kết quả mạnh hơn như sau:

Nếu $P(x) = (x^2-7x+6)^{2n} + 13 $ có thể phân tích thành tích của 2 đa thức Q(x), S(x) với hệ số nguyên thì Q(x) và S(x) đều có bậc 2n.

Từ đây với n > 1 bài toán trở nên hiển nhiên, còn với n = 1 ta có thể kiểm tra trực tiếp rằng $(x^2-7x+6)^2 + 13 $ bất khả quy.

Thật vậy, giả sử P(x) = Q(x).S(x).

Gọi $x_1, x_2, ..., x_{4n} $ là các nghiệm phức của P(x) thì sẽ là tích của các thừa số $(x-x_i) $. Đánh số lại nếu cần, ta giả sử $Q(x) = (x-x_1)(x-x_2)...(x-x_k) $ với $1 \le k < 4n $.

Ta có $((x_i-1)(x_i-6))^{2n} = -13 $. Từ đây suy ra $|(x_i-1)(x_i-6)| = 13^{\frac{1}{2n}} $. (*)

Mặt khác $(1-x_1)...(1-x_k) = Q(1) $ nguyên nên $|(1-x_1)...(1-x_k)| $ nguyên. Tương tự $|(6-x_1)...(6-x_k)| $ nguyên.

Từ đây suy ra $m = |(x_1-1)(x_1-6)(x_2-1)(x_2-6)...(x_k-1)(x_k-6)| $ nguyên.

Nhưng theo (*) thì $m = 13^{k/2n} $

Suy ra k = 2n. Vậy Q(x), S(x) đều phải có bậc là 2n.

Tôi có cảm nhận rằng đa thức P(x) là bất khả quy, tuy nhiên cần thời gian để kiểm tra chắc chắn.

Lời giải trên đây dựa trên ý tưởng của 2 bài toán kinh điển sau:

1. Tìm tất cả giá trị n sao cho đa thức $x^n + 4 $ có thể phân tích thành tích của hai đa thức khác hằng số với hệ số nguyên.

2. (IMO 1993) Chứng minh rằng với mọi n> 1, đa thức $x^n - 5x^{n-1} + 3 $ bất khả quy.
[RIGHT][I][B]Nguồn: MathScope.ORG[/B][/I][/RIGHT]
 

thay đổi nội dung bởi: namdung, 04-01-2014 lúc 08:35 AM
namdung is offline   Trả Lời Với Trích Dẫn
The Following 8 Users Say Thank You to namdung For This Useful Post:
12121993 (04-01-2014), DaiToan (04-01-2014), DogLover (05-01-2014), hakudoshi (04-01-2014), huynhcongbang (04-01-2014), luugiangnam (05-01-2014), quangvinht2 (04-01-2014), thaygiaocht (04-01-2014)
Old 05-01-2014, 10:43 AM   #11
hung_020297
+Thành Viên+
 
Tham gia ngày: Mar 2012
Đến từ: thanh hoa roi
Bài gởi: 45
Thanks: 15
Thanked 1 Time in 1 Post
Nếu mà làm được đến đa thức bậc 2 thì có được điểm ko ạ
[RIGHT][I][B]Nguồn: MathScope.ORG[/B][/I][/RIGHT]
 
hung_020297 is offline   Trả Lời Với Trích Dẫn
Old 05-01-2014, 11:10 AM   #12
kien10a1
+Thành Viên+
 
kien10a1's Avatar
 
Tham gia ngày: Feb 2011
Đến từ: Vĩnh Yên- Vĩnh Phúc
Bài gởi: 371
Thanks: 43
Thanked 263 Times in 153 Posts
Gửi tin nhắn qua Yahoo chát tới kien10a1
Trích:
Nguyên văn bởi namdung View Post

Tôi có cảm nhận rằng đa thức P(x) là bất khả quy, tuy nhiên cần thời gian để kiểm tra chắc chắn.
Em thấy như thầy Nam Dũng phân tích, thì ta chắc chắn có $\left | Q(1)Q(6) \right |=13 $
Vậy thì mọi chuyện xong rồi chứ nhỉ, chú ý thêm $ Q(1)-Q(6) \vdots 5 $ thì ta suy ra ngay không có đa thức Q hệ số nguyên thỏa mãn.
[RIGHT][I][B]Nguồn: MathScope.ORG[/B][/I][/RIGHT]
 
__________________
Quay về với nơi bắt đầu
kien10a1 is offline   Trả Lời Với Trích Dẫn
Old 05-01-2014, 01:46 PM   #13
vickyjustice
+Thành Viên+
 
Tham gia ngày: Sep 2013
Bài gởi: 10
Thanks: 0
Thanked 4 Times in 3 Posts
Xet P(x+1) roi nhan xet neu phan tich dc thoa de bai thi co 2 da thuc bac 2 vo nghiem, he so tu do la 13 nen he so tu do cua 1 trong 2 da thuc do phai =1, vay chi co 3 truong hop: $x^2+1 $, $x^2+x+1 $, $x^2-x+1 $, thay nghiem phuc thay vo ly
[RIGHT][I][B]Nguồn: MathScope.ORG[/B][/I][/RIGHT]
 

thay đổi nội dung bởi: vickyjustice, 05-01-2014 lúc 01:49 PM
vickyjustice is offline   Trả Lời Với Trích Dẫn
The Following User Says Thank You to vickyjustice For This Useful Post:
thaygiaocht (05-01-2014)
Trả lời Gởi Ðề Tài Mới

Bookmarks

Ðiều Chỉnh
Xếp Bài

Quuyền Hạn Của Bạn
You may not post new threads
You may not post replies
You may not post attachments
You may not edit your posts

BB code is Mở
Smilies đang Mở
[IMG] đang Mở
HTML đang Tắt

Chuyển đến


Múi giờ GMT. Hiện tại là 08:14 PM.


Powered by: vBulletin Copyright ©2000-2024, Jelsoft Enterprises Ltd.
Inactive Reminders By mathscope.org
[page compression: 92.10 k/106.32 k (13.38%)]